Torts Final

¡Supera tus tareas y exámenes ahora con Quizwiz!

On a dark night, Jim who is 40 years old is walking in a city park that winds along the coastline on a bluff above the water. While walking on a path adjacent to a stone wall marking the edge of the bluff, the ground gives way, and Jim falls to the bottom of the bluff, suffering physical harm. A city ordinance prohibits entry into the park from dusk until dawn, but no physical barrier prevents entry during that time nor is there a sign prohibiting entry. Jim sues City in negligence. Which of the following statements about Jim's status and City's duty is correct? A Jim is a trespasser to whom City owes a duty of reasonable care. B Jim is a trespasser to whom City owes no duty of care. C Jim is a licensee to whom City owes a duty to render the premises reasonably safe. D Jim is a business invitee to whom City owes a duty to render the premises reasonably safe.

A Correct. At the time of his injury, Jim was a trespasser as, pursuant to the ordinance, he was not supposed to be in the park after dark. This question asks about the duty of care owed to a trespasser. A trespasser is a person who has entered another's land without consent or other legal privilege. A possessor of land owes a duty of reasonable care to entrants of the land with regard to natural conditions of the land that pose a risk of harm. Restatement (Third) Torts § 51.

Rosie and Vincent do not know each other and were camping independently in a heavily forested campground. Each one had a campfire, and each one negligently failed to ensure that the fire was extinguished upon retiring for the night. Due to unusually dry forest conditions and a stiff wind, both campfires escaped their sites. The two fires, burning out of control, eventually joined together and engulfed Company's hunting lodge, destroying it. Either fire alone would have destroyed the lodge. In a lawsuit brought by Company against both Rosie and Vincent alleging negligence, which of the following statements is correct regarding factual causation? A Each of Rosie's and Vincent's negligence is a factual cause of the destruction of Company's hunting lodge. B Neither Rosie's nor Vincent's negligence is a factual cause of the destruction of Company's hunting lodge. C Rosie's negligence is the sole factual cause of the destruction of Company's hunting lodge. D Vincent's negligence is the sole factual cause of the destruction of Company's hunting lodge.

A Correct. When multiple acts occur and each alone would have been a factual cause in the absence of the other act, each act is a factual cause of the harm. This is a question about factual causation when two independent acts of negligence join together to create one harm where either independent act alone would have been sufficient in creating the harm. Restatement (Third) Torts § 27.

Doctor while conducting an operation on plaintiff's left eye decides to examine the rest of plaintiff's face and discovers that plaintiff has a deviated nasal septum. At the time of the discovery the plaintiff was under the effect of general anesthesia and unconscious. The doctor decides to correct the deviated septum while the plaintiff rather than do it separately which would require another operation. Which of the following is true? A) Doctor likely committed a battery. B) Doctor will not be liable unless he performed the nasal septum operation negligently. C) Doctor's may assume consent from patients who are under general anesthesia to perform medically warranted procedures. D) Doctor likely committed an assault and battery.

A. This is a classic case of battery in tort law. Battery is a dignitary tort and is based on the individual's right to permit or not allow contact with his or her body. Even though the doctor acted with "good" intentions, motive is not the same as intent. Here the doctor committed nonconsensual contact to the person of another because presumably he only had consent to operate on the patient's eye and perform any contact associated with that. If a reasonable person would find the extra contact to be harmful or offensive then battery would be actionable. Even though the damage may be minimal many intentional tort suits survive on nominal damages. The purpose of this is that the decisions become judicial statements on what is permitted and not permitted under the law.

Hoda, a diabetic, required special orthopedic shoes. Podiatrist negligently fitted Hoda with her orthopedic shoes. The improperly fitted shoes caused blistering on Hoda's feet. Hoda saw Dr. Primary about the blistering, but Dr. Primary decided nothing need be done. However, because of her diabetic condition, Hoda developed gangrene in one of her feet and had to have her leg amputated. Hoda sued Podiatrist for his negligence in fitting her with the shoes, seeking to recover damages for the amputation of her leg. Podiatrist contends that if Dr. Primary had not been negligent in failing to take prompt action to treat Hoda's blistering, Hoda would not have lost her leg and Podiatrist therefore should not be liable to Hoda for the loss of her leg. Which of the following statements is correct? A Podiatrist should not be liable to Hoda for the loss of her leg because Dr. Primary's negligence was a superseding cause. B Podiatrist may be liable to Hoda for the loss of her leg because Dr. Primary's negligence was a foreseeable intervening event. C If Podiatrist is liable to Hoda for the loss of her leg, then Dr. Primary cannot also be liable. D If Dr. Primary is labile to Hoda for the loss of her leg, the Podiatrist cannot also be liable.

B Correct. A person may be liable to another when their negligence was a foreseeable intervening event. This question asks about the liability of a negligent defendant when there is an intervening negligent act that contributes to the plaintiff's harm.

Ellie, who was employed as an asbestos insulation installer for many years, is killed when a defect in a ground-fault interrupter (GFI) in her bathroom fails. A GFI is designed to protect a person from a dangerous or lethal electrical shock hazard by breaking the electrical current if the electrical device comes into significant contact with water. The GFI in Ellie's bathroom failed to break the electrical circuit after an electrical razor fell in the bathtub, resulting in Ellie's electrocution and death. The autopsy reveals that Ellie died from the electrocution. It also shows that she was in the early stages of mesothelioma, which would have killed her eventually. Mesothelioma is a rare and almost invariably fatal form of cancer that studies have established is caused only by exposure to asbestos. In a lawsuit against the manufacturer of the GFI, what is the factual cause of Ellie's death? A The defect in the GFI is not a factual cause of Ellie's death because she was suffering from a fatal form of cancer at the time and would have died from it eventually. B The defect in the GFI is a factual cause of Ellie's death. C The defect in the GFI and the asbestos in the insulation were both a factual cause of Ellie's death. D Neither the GFI nor the asbestos in the insulation was a factual cause of Ellie's death.

B Correct. For the GFI defect, Ellie would not have died from electrocution when she did.

B suddenly and violently attacks A with a machete. A evades the machete and kicks B in the stomach causing B to stumble down some stairs and to collide with C who unknown to A was coming up the stairs. The collision causes C to suffer a torn ligament in his knee. If A raises the self-defense would A be liable to C? A) A would be liable to C because C never used any force against A so A had no privilege which permitted force against C. B) A would not be liable to C because A acted reasonably in defending herself against B. C) A would be liable to C because A had to absolutely ensure her use of force against B would not cause harm to anyone besides B. D) A would not be liable even if A acted unreasonably in defending herself against B.

B Correct. If a bystander is hurt by a person's use of defensive force against another then there is no liability as long as the defensive force was reasonable.

A light artist lives in a densely populated suburban area. Light artist's home is surrounded by other houses. Chain link fences separate the houses. Every night light artist goes into his back yard and intentionally shines lights of all different colors on the lawns of her neighbors as part of a light show. The neighbors do not appreciate the lights on their property because they find them distracting and annoying. In an action for trespass to land against light artist will neighbors likely win or lose? A Neighbors will likely win because light artist intentionally causes light to enter on to their real property. B Neighbors will likely lose because the light is not tangible. C Neighbors will likely win if the light decreases their property value. D Neighbors will lose because the light causes no actual harm.

B Correct. Trespass to land requires a tangible entry onto the land of another and light is not considered tangible.

A pointed a gun at B scaring B, and without authority or justification, A ordered B into a room and locked the door for 2 hours. B was awake the whole time and worried about what would happen until A released him unharmed. When A initially pointed gun at B, B was standing next to C, and C dove for cover thinking the gun was pointed at her. In diving for cover C collided violently with D causing D to spill coffee on E's computer, completely destroying the computer. E is no longer interested in retaining title to the computer but simply wants A to pay the market value of the chattel at the time the chattel was destroyed and to transfer title of the computer to A. Who may maintain an action against A? A) Only B but not C, D and E may maintain an action against A B) Only B and C but not D and E may maintain an action against A C) Only B, C and D but not E may maintain an action against A D) B, C, D and E may all maintain actions against A

C. Correct. It correctly applies the doctrine of transferred intent and the rule of extended liability in intentional tort. Transferred intent operates whenever the tort intended and the tort committed are within the original writ of trespass (Trespass to Land, Trespass to Chattel, False Imprisonment, Assault and Battery). Whenever a tortfeasor intends any one of the five but ends up committing any other of the five torts then all the torts committed are deemed to have the intent prong satisfied. For example in the above scenario A intended to falsely imprison B, but while committing false imprisonment he also caused B to get scared. Therefore A has committed both an assault and a false imprisonment against B. Additionally, not only does intent transfer between torts but it transfers between people as well. So even though A was intending only to falsely imprison B, he also caused C to get scared and therefore has committed assault against C. Similarly, even though A's only intent was to falsely imprison B, his tortious conduct scared C and resulted in a violent collision with D (harmful or offensive contact) and as a result D has an actionable battery claim against A. However A's intent to falsely imprison B caused the complete destruction of E's computer (a chattel). Now rather than suing for the harm caused to the chattel as in the previous question E is seeking to have A pay the entire market price of the computer when it was destroyed. This means that rather than trespass to chattel against A, E is seeking to recover pursuant to conversion. Conversion unlike trespass to chattel is not one of the torts within the original writ of trespass and therefore transferred intent will not support E's action against A. Also relevant to this hypothetical is the rule of limitless causation in intentional tort. A tortfeasor is usually liable for all the harm caused by their tortious conduct whether or not the harm was foreseeable. While this rule will help C and D recover it will not help E recover here. The reason is a little complicated. Because transferred intent does not apply, in order to recover E would have to prove that A was at least substantially certain that trying to imprison B would cause the destruction of E's computer. Because A's intent to imprison does not transfer to a conversion action and the facts do not indicate that A was substantially certain to destroy E's computer when he tried to imprison B, the intent prong of the conversion suit would fail and E would not be able to recover against A for the loss of the computer. It is important to remember that even though an intentional tortfeasor is potentially liable for all the harm they cause, recovery for harm is possible only if there is a maintainable cause of action to recover for that harm. In this case E cannot maintain a conversion cause of action.

Charles is a guest in Heights Hotel. The bathroom in the hotel includes a shower, which is protected by a sliding door made of ordinary glass. While taking a shower, Charles trips and falls on the glass door, causing it to shatter. The shards of glass cut Charles causing serious injuries. It is a standard practice among hotels to use shatterproof, safety glass rather than ordinary glass at shower enclosures. In a negligence action brought by Charles against Heights Hotel, what effect, if any, will Hotel's departure from the industry custom have on Hotel's negligence? A Heights Hotel's departure from this custom is evidence of its negligence. B Heights Hotel's departure from this custom is conclusive proof of its negligence. C Evidence of Heights Hotel's departure from this industry custom is legally irrelevant to the issue of its negligence. D Despite evidence of that Heights Hotel disregarded custom, Heights Hotel may not be subject to liability in negligence because Charles tripped and fell, causing his injuries.

A Correct. A Defendant's departure from custom is evidence that the defendant was negligent, but such departure does not require a finding of negligence. Restatement (Third) § 13.

Driver negligently runs a red light and hits Bob's daughter who suffers serious injury. Bob sees the accident from the sidewalk and suffers serious emotional harm. Bob sues Driver in negligence for his emotional injuries. Can Bob recover from Driver? A Bob can recover from Driver because Driver negligently caused a sudden serious bodily injury to Bob's daughter and Bob perceived the event contemporaneously. B Bob cannot recover from Driver because recovery for negligence that causes pure emotional injury is not permitted. C Bob cannot recover from Driver because Bob was not physically impacted by the accident. D Bob can recover from Driver because his close family member was injured.

A Correct. A defendant who negligently inflicts sudden serious bodily injury to another is subject to liability for serious emotional harm caused to a person who contemporaneously perceived the accident and who is a close family member of the injured person. Restatement (Third) Torts § 48.

Which of the following statements about factual and proximate causation is correct? A Proximate causation determinations in negligence actions establish whether to shield a defendant from liability even though the defendant's breach of the standard of care was a factual cause of the plaintiff's harm. B Proximate causation determinations are unique to negligence actions and are not made in intentional torts or strict liability actions. C Legal causation and factual causation are synonymous terms. D Factual causation and proximate causation are synonymous terms.

A Correct. Causation in a negligence action is comprised of factual causation (which is also called but for causation or factual causation) as well as legal causation (which is often termed proximate causation). Legal causation shields a defendant from liability even though they are a factual cause of the complained of harm.

Drug Co. negligently manufactures and sells a defective contraceptive pill, which fails to prevent Yolanda's pregnancy. Yolanda gives birth to Child A, who grows up and gives birth to Child B. Child B grows up to be a motorcycle enthusiast, who negligently causes a motorcycle accident that injures Charles. Charles seeks to recover from Drug Co. in a negligence action, alleging that Drug Co.'s negligence was a factual cause of Charles's injury. Will Charles's negligence action likely succeed or fail? A Charles is correct that Drug Co. is a factual cause of Charles's injury, but his negligence action will likely fail for lack of proximate cause. B Charles is incorrect that Drug Co. is a factual cause of Charles's injury and therefore cannot be subject to liability in negligence to Charles. C Charles is correct that Drug Co. is a factual cause of Charles's injury, and therefore his negligence action against Drug Co. will likely succeed. D Charles is incorrect that Drug Co. is a factual cause of Charles's injury, but his negligence action against Drug Co. may still succeed.

A Correct. Even though Drug Co. is a factual cause of Charles's injuries, his injuries are likely too remote from Drug Company's negligence in order for its negligence to be a proximate cause of his injuries. This question asks about proximate cause determinations in a products liability action grounded in negligence.

Law student is angry at Torts professor because professor made student work and professor is always so prepared in class. As professor is walking to his car in his perfectly matching designer shoes, "I am going to hit you with this book for being so prepared." Student did not intend to hit him but just to scare him by swinging the book close to his head and making him flinch. Unfortunately student misjudged the length of the book and the book struck professor's head after professor flinched. Are the student's actions assault and/or battery? A The students actions are both an assault and battery. B The students actions are neither an assault nor a battery. C The students actions are an assault but not a battery. D The students actions are a battery but not an assault.

A Correct. Here the student intended make the professor afraid of imminent contact with the book on his head. Being hit in the head with a book is clearly harmful or offensive. In other words the student committed an assault. Student had the intent to cause the professor fear or apprehension of imminent harmful or offensive contact and the professor's flinch indicates that he did experience some form of fear or apprehension. Therefore assault occurred. Battery also occurred. Even though the Student did not have the intent to cause harmful or offensive contact with the professor, harmful or offensive contact is exactly what ended up happening when the professor was unintentionally hit in the head. The lack of intent to hit does not prevent a battery claim however. This is because the intent to assault will transfer to satisfy the intent requirement of battery via the doctrine of transferred intent. Transferred intent operates whenever the tort intended and the tort committed are within the original writ of trespass (Trespass to Land, Trespass to Chattel, False Imprisonment, Assault and Battery). Whenever a tortfeasor intends any one of the five but ends up committing any other of the five torts then all the torts committed are deemed to have the intent prong satisfied.

Joe has just finished eating his lunch and is walking alone down the sidewalk in front of a restaurant when he suddenly suffers a severe asthma attack. Waiter, who works at the restaurant but is on a break at the time, sees Joe through a window and realizes that Joe is suffering an asthma attack. Nonetheless, Waiter chooses to do nothing to help Joe. As a result, the appropriate medical care for Joe is delayed, causing Joe serious physical injuries that could have otherwise been avoided. Joe sues Waiter in negligence for his injuries that result from the delay in his receiving medical care. May Waiter be liable to Joe in negligence? A Waiter may not be subject to liability in negligence because Waiter had no affirmative duty to render aid or call for assistance for Joe. B Waiter may be subject to liability in negligence for failing to render aid or call for assistance for Joe because Waiter realized that Joe was in need of prompt medical attention. C Waiter may be subject to liability in negligence for failing to render aid or call for assistance for Joe, but only if Waiter knew that no one else was present to call for assistance. D Waiter cannot be subject to liability in negligence because Waiter's failure to act was not the factual cause of Joe's injuries.

A Correct. Here, Waiter did not create any risk of harm to Joe. This question asks about one's duty to act affirmatively in negligence. The general rule is that a defendant has no affirmative duty to act unless the defendant has created a risk of physical or emotional harm to another. Restatement (Third) Torts § 37.

For almost 2 centuries Landowner's family has owned a 2 miles stretch of coastal land just landward of a public beach. As the land has been passed down from generation to generation landowner's family has always allowed the public to walk on the land to get beach access even though there are many conspicuous signs saying private property on the land. The newest descendant to own the land has decided that he no longer wants anyone to access the land without permission. One day after this, beachgoer, as other beachgoers have done for centuries walks on the land to get to the beach. Did beachgoer trespass? A No, because a custom existed which permitted entry onto the land that would otherwise be trespass. B Yes, because customary consent may be unilaterally withdrawn without notice. C Yes, because the private property signs meant that entry onto the land was trespass. D No, because once you allow access pursuant to custom it cannot be revoked.

A Correct. If a custom exists which permits entry which otherwise would be a trespass then before entry is actionable the landowner must affirmatively provide notice that the consent is withdrawn.

Connie is carelessly changing lanes in her car when her car is struck by Bill who is reaching for his phone, which has fallen under the passenger side seat while he was driving. Adrian, who is in the vicinity on her motorcycle waiting patiently at the intersection for the light to turn green, is struck by Connie's car and suffers serious physical injury as well as damage to her bike. She sues both Connie and Bill for their negligence. The jury determines that Adrian's damages are $100,000 and concludes that Bill and Connie are both liable. It assigns 30% responsibility to Bill, and 70% responsibility to Connie. In a jurisdiction that subjects defendants to joint and several liability, what amount, if any, is Adrian entitled to recover from Bill and/or Connie? A If he chooses, Adrian may recover $100,000 from Bill alone. B Adrian is entitled to recover only $30,000 from Bill and only $70,000 from Connie for a total of $100,000 C Adrian is entitled to recover $100,000 from both Bill and Connie for a total of $200,000. D If he chooses, Adrian may recover up to $30,000 from Connie and $70,000 from Bill.

A Correct. Joint and several liability means that each tortfeasor is 100% responsible for the entire judgment and the plaintiff is free to recover her full judgment from any one or combination of them. Therefore, Adrian can recover the entire judgment from Bill alone.

In a negligence action brought by Plaintiff Betty against Defendant Alice and Defendant Carl, the jury apportions liability based on the negligence of the parties as follows: Betty is 20% at fault; Alice is 60% at fault; and Carl is 20% at fault. The jury also determines that Plaintiff Betty sustained $100,000 in compensatory damages. In a pure comparative fault jurisdiction that provides that defendants are severally liable, what amount, if any, may Betty recover? A Betty may recover up to 60% of $100,000 from Alice and up to 20% of $100,000 from Carl. B Betty may recover up to 60% of $80,000 from Alice and up to 20% of $80,000 from Carl. C Betty may recover 60% of $80,000 from Alice or Carl or a combination of both. D Betty may not recover at all.

A Correct. Plaintiff can recover 60% of the $100,000 from Alice and 20% of $100,000 from Carl. This question asks about the several liability of defendants in a pure comparative fault jurisdiction. In jurisdictions with such schemes, the defendant is only responsible for the percentage of the damages that are attributable to it.

George, a concessionaire at a sports stadium, negligently collided with Fanny, a fan attending the game, knocking her to the ground. Fanny had an asymptomatic herniated disc in her back that results in her suffering serious back injury and pain as a result of George's collision with her. In a negligence action brought by Fanny against George, may George be subject to liability for Fanny's injuries? A That Fanny could be injured was foreseeable from George's negligence and therefore George can be subject to liability. B George cannot be held liable for Fanny's injuries because he could not have foreseen that she was suffering from an asymptomatic herniated disc. C George may be subject to liability for the collision itself, but not for the extensive back injuries because the extensive back injuries were not foreseeable. D George may only be held liable to Fanny for injuries that he intended to cause.

A Correct. Pursuant to the eggshell skull plaintiff rule, a plaintiff's physical injuries are foreseeable.

Toddler, a two-year-old child suffering from a bacterial infection, received a routine childhood vaccination with a vaccine manufactured by Vaco. Shortly after receiving the vaccine, Toddler spiked a very high fever, went into respiratory arrest, and died. Toddler's estate sues Vaco in negligence. At trial, Toddler's estate provides competent evidence that Toddler died due to a combination of the infection and the vaccine and, if either toddler had not had an infection or if Toddler had not been vaccinated, death would not have ensued. Regarding the factual causation element of the negligence action, has Toddler's estate satisfied its burden of production to prove that the vaccine was a factual cause of Toddler's death? A Toddler's estate has satisfied its burden of production to show that the vaccine was a factual cause of Toddler's death. B Toddler's estate has not satisfied its burden of production to show that the vaccine was a factual cause of Toddler's death C Toddler's estate cannot satisfy its burden of production to show that the vaccine was a factual cause of Toddler's death because the infection contributed to Toddler's death. D Toddler's estate cannot satisfy its burden of production to show that the vaccine was a factual cause of Toddler's death because the infection was the sole factual cause of Toddler's death.

A Correct. See Restatement (Third) Torts § 26 and illustration 4, upon which this problem is based. The estate's evidence that Toddler died due to a combination of the infection and the vaccine coupled with proof that either toddler had not had an infection or if Toddler had not been vaccinated, death would not have ensued, is sufficient to prove factual causation.

On Friday, employee is credibly threatened by coworker. Coworker says to employee, "I will kill you next Friday if you show up to work then." Employee and three friend's go to Coworker's house the day after Coworker threatened employee and beat Coworker to such an extent that the Coworker will be hospitalized long term and incapable of carrying out his threat. May the employee successfully assert the privilege of self-defense in a battery suit by Coworker against Employee? A No, because the threat by coworker was not one which required immediate action. B Yes, because Coworker credibly threatened employee with death so employee could use up to lethal force in defending himself. C Yes, because employee can preemptively prevent the threat from materializing. D No, because the person asserting self-defense must wait for the attacker to begin attacking before defending.

A Correct. Self-defense or the privileged use of force against another to prevent harm to oneself requires the harm be imminent or immediate or that the imminent or immediate use of force is necessary to prevent future harm. Here the harm is not imminent as Coworker threatened harm in a week. Additionally, there are other avenues such as reporting Coworker to the authorities which are preferable here to the use of force "vigilante justice" by employee.

Shopper is picking up a few items at Grocery Store on her way home from work. As she enters the produce section of Grocery Store, she sees Plaintiff lying on the floor, holding her obviously injured hip. Right next to Plaintiff, Shopper observes a dry, gritty, almost entirely black, squishy banana lying on the floor. In Plaintiff's negligence action against Grocery Store alleging Grocery Store's negligence, Shopper testifies to the condition of the banana when she saw it as it lay next to Plaintiff who was on the floor as well. What is the best characterization of Shopper's testimony presented in the trial brought by Plaintiff against Grocery Store for its alleged negligence? A Shopper's testimony is circumstantial evidence of Grocery Store's negligence and requires the factfinder to draw an inference regarding Grocery Store's breach of the standard of care. B Shopper's testimony is direct evidence of Grocery Store's negligence and requires the factfinder to draw an inference regarding Grocery Store's breach of the standard of care. C Shopper's testimony is circumstantial evidence and cannot be used to establish Grocery Store's breach of the standard of care. D Shopper's testimony is direct evidence of Grocery Store's negligence and cannot be used to establish Grocery Store's breach of the standard of care.

A Correct. Shopper's evidence is circumstantial evidence from which the factfinder is asked to infer that Grocery Store breached its standard of care. The fact that the banana was much trampled on, black all over with very little yellow, gritty, sticky, and yucky indicates that the banana had been on the floor for a long time. If the banana had been on the floor for a long time, one can infer that Grocery Store did not keep its premises in a reasonably safe condition.

A state statute requires that the operator of a truck that becomes disabled on a highway promptly put out a warning sign at least 100 feet behind the truck. When a deflated tire disables Chuck's truck, he promptly places out a warning sign right next to the truck rather than at the 100-foot distance. Alice, approaching Chuck's truck from behind, does not see Chuck's warning sign until it is too late for her to stop. Her car strikes the rear of Chuck's truck, and she is injured in the collision. Alice sues Chuck in negligence, and the evidence at trial shows that Alice would have been able to stop in time had the warning sign been set at the 100-foot distance as required by statute. What effect, if any, does Chuck's violation of the statute have on his liability for negligence? A Chuck's violation of the statute may be used to establish his negligence, because the basic purpose of the statute is to prevent accidents of this type and Alice is in the group of persons the statute is designed to protect. B Chuck's violation of the statute is evidence of res ipsa loquitur because the basic purpose of the statute is to prevent accidents of this type and Alice is in the group of persons the statute is designed to protect. C Chuck's violation of the statute has no effect on his liability for negligence because the statute cannot be used to establish the standard of care in Alice's negligence action against Chuck because the statute was not designed expressly to provide a private right of action in negligence. D Chuck's violation of the statute has no effect on his liability for negligence because the standard of care in negligence must be established by proving what the reasonable prudent person in the same or similar circumstances would have done.

A Correct. This response correctly provides the standard for using a statute to establish the standard of care in negligence. Restatement (Third) Torts § 14. This question asks about using a statute's violation to establish the standard of care in a negligence action.

Shopper insulted store clerk by saying that the store clerk had a disgusting body odor. Shopper intended to insult and embarrass store clerk and said it loud enough for the fellow shoppers in the vicinity to hear. Store clerk was mortified when other shoppers laughed. As a result of the insult, store clerk fainted and hit his head on a display and suffered a concussion. Assume that the shopper intentionally, impermissibly and offensively touched the store clerk while he made the comment but the store clerk was only mildly embarrassed and suffered no additional physical injuries due to the touch. Will store clerk be able to recover? A Store clerk may be able to recover for both the impermissible contact and the mild embarrassment. B Store clerk may not recover for the emotional harm because it was not severe but may recover for the impermissible contact. C Store clerk may not recover for either the emotional harm or the impermissible contact. D Store clerk may not recover for the emotional harm unless the touching was considered extreme and outrageous conduct but may recover for the impermissible touching.

A The intentional, impermissible and offensive touching will constitute a battery. And the tort of battery unlike the tort of IIED is actionable on only nominal damage as it is a dignitary tort . The requirement of outrageous and extreme conduct and severe harm are only conditions precedent to recovery if the cause of action is Intentional Infliction of Emotional Distress. Here the store clerk will likely have an actionable battery and if the store clerk elects to sue in battery then any and all associated emotional harms are recoverable. It is only when the cause of action is IIED that the requirements of outrageous conduct and severe harm must be satisfied before recovery of emotional harm is permitted.

Father was severely beaten by robber and is bringing a claim for assault and battery against robber. Unknown to robber, daughter witnessed father being beaten and suffered emotional distress as a result. If daughter brings a claim for intentional infliction of emotional distress (IIED) against robber, may daughter recover? A Daughter may not recover because robber did not know she witnessed the father's beating. B Daughter may recover only if she experienced bodily injury due to the emotional distress. C Daughter may not recover because she did not suffer bodily injury due to the emotional distress. D Daughter may recover even if robber was unaware of her presence.

A The rule by which bystanders may recover pursuant to IIED for tortious conduct directed at another is that the tortfeasor must have known or have been substantially certain the bystander witnessed the tortious conduct directed at another person. The reason for this is that IIED is not one of the torts within the original writ of trespass and so the intent does not transfer from the tortious conduct against the father so as to support the intent prong of the daughter's IIED claim in this scenario. In order for transferred intent to apply the tort intended and the tort committed must be within the original writ of trespass (Trespass to Land, Trespass to Chattel, False Imprisonment, Assault and Battery).

About two years ago, Client was a patient of Ophthalmologist. Client alleges that during the time that he was Ophthalmologist's patient, Ophthalmologist improperly performed LASIK surgery on Client's eyes. The LASIK surgery was supposed to correct Client's poor vision. Instead, the surgery was performed skillfully, but resulted in blindness in both eyes, a risk about which Client was never informed and a reasonable person would not have known. If Client had known of the risk of blindness, he would not have opted for the surgery. Client hires Attorney Matthew to sue Ophthalmologist for medical malpractice. Attorney Matthew fails to file the complaint to initiate the cause of action against Ophthalmologist within the applicable statute of limitations. As a result, the court dismisses Client's case against Ophthalmologist. Client then hires Attorney Becky to represent him in a cause of action against Attorney Matthew for legal malpractice. In the legal malpractice action brought by Client against Attorney Matthew, how will the professional standard of care owed by Attorney Matthew need to be established? A Client will have to establish, by introducing expert testimony, that Attorney Matthew failed to perform as the reasonable prudent attorney in handling Client's medical malpractice action against Ophthalmologist. B Client will be able to establish that Attorney Matthew failed to perform as the reasonable prudent attorney in handling Client's medical malpractice action without expert testimony regarding the standard of care. C Client will be able to establish the standard of care in the legal malpractice action against Attorney Matthew by applying a rule of law. D The factfinder will determine the standard of care in this legal malpractice action without evidentiary guidance.

A Correct. This question asks about establishing the standard of care in a professional negligence action. The general rule is that the professional standard of care must be established by expert testimony.

Furniture delivery person knocks on homeowner's door and announces, "We have a side table here for delivery." Homeowner lets the delivery person in the house and leads her to the dining room where the side table is to be assembled. Homeowner then leaves the area to go upstairs. While assembling the table in the dining room, the delivery person hears what she thinks is a puppy barking playfully in the basement of the house. While homeowner is upstairs the delivery person walks through the house looking for the basement door, finds the door and enters the basement and finds a toy dog barking. The delivery person laughs and returns to the dining room to assemble the table. While returning to the table the delivery person leaves a small shoe stain on the carpet in the basement. Has the delivery person trespassed? A) Yes, because she likely exceeded the scope of the permission granted to her by the homeowner to enter the house. B) No, because once the homeowner gave permission to enter the house the delivery person was free to go anywhere in the house. C) Yes, because the delivery person caused actual damage when she stained the carpet. D) No, if the reason the furniture delivery person entered the basement was to play with the puppy.

A. Trespass to land is actionable if the defendant exceeds the scope of the permission to enter the land. Many examples of this are common with meter readers, delivery workers who are given permission to enter land for a specific purpose. If they exceed the scope of the permission granted then a permissible entry becomes a trespass. Here the facts do not indicate the delivery person was given permission to roam the entire house, rather she was given permission to be in the dining room and to do what was necessary to complete the installation. This likely did not encompass roaming the house and entering the basement to play with a dog. Another classic example of this is when customers enter a store to shop. Clearly they are invited into the shopping area and cannot be trespassing if they remain there. However, if once in the store they enter for example a locked area of the store with a sign saying, "Employees Only," then they morph from permissible entrants to trespassers.

Pedestrian is walking down the street when she notices person about to be hit with a baseball bat by screaming attacker. Pedestrian is a martial artist with years of experience and disarms attacker before he can harm person. In the process of disarming the screaming attacker she twists his arm and pins his arm, throws him to the street and holds him there until the police arrive. Does attacker have a viable suit against pedestrian for battery? A Yes, because pedestrian intentionally caused harmful or offensive contact to the screaming attacker. B No, if pedestrian's actions were reasonable C Yes, because pedestrian does not have a right to interfere in the affairs of others. D No, because a person may use any force to defend another person from tortious conduct.

B Correct. A person may use reasonable force to defend another person from an attack that appears to threaten imminent harm. This privilege operates exactly like the privilege of self defense.

A and B are on a first date. A says to B, "I would really like to kiss you." B responds by shrugging her shoulders. A then kisses B. If B sues A for battery which of the following if true would be A's best defense? A A honestly believed B consented by shrugging her shoulders. B A reasonably believed B consented by shrugging her shoulders. C A did not know what the shrug meant but kissed B anyway. D A only intended to make B happy when he decided to kiss her.

B Correct. Consent is an objective standard. If a person reasonably believes another consented to an act then consent exists even if subjectively the other person did not consent.

E and F have been friends for a long time. For years they have played a game where they hit each other on the shoulder. The game starts by one hitting the other and then the other returns the hit slightly harder. This sequence continues until one person is hit hard enough and "surrenders." One day F hits E on the shoulder in an attempt to start the game. E sues F for battery. What is the likely result of this suit? A E will win because F hit him. B F will win because E consented to being hit. C E will win because he never consented to this specific episode of contact. D F will win because once you consent to being hit you can never recover for being hit in battery.

B Correct. Consent may be implied from a course of conduct between the parties. Here these two friends have engaged in similar conduct for years. Unless something changed or was said there was implied consent for similar contact between the two and therefore the battery suit would be unsustainable.

Soccer player asks permission to use soccer field from field owner. Field owner gives consent for soccer player to use soccer field between 4:00 pm and 6:00 pm. After using the field a few times between 4 and 6 pm, soccer player notices no one uses the field after 6:00 pm and decides to use the field even after the time consented to by the owner. May owner sue soccer player for trespass to land due to soccer player's use of the field after 6:00 pm? A No, because field owner consented to use of the field. B Yes, because soccer player exceeded the scope of the consent granted. C No, unless the soccer player harmed the field when using it after 6:00 pm. D Yes, if owner had previously cautioned the soccer player at least once before when soccer player used the field after 6:00 pm.

B Correct. Consent to permit conduct which otherwise would be tortious may be limited in scope. For example here the consent to allow use of the property was limited to between 4:00 and 6:00 pm. Any other use would constitute a trespass to land. Here the scope of the consent was limited to use between 4:00 and 6:00 pm and the soccer player exceeded the scope of that consent by using it at other times.

Customer left her waterproof coat on a coat rack at the entrance to a restaurant. Thief decided to steal the coat and keep it for himself. After 3 hours the thief had a change of heart and figured out where customer lived. Thief returned the coat to customer's address unharmed. However because of Thief's actions Customer had to leave the restaurant without her coat. As she was walking home it rained suddenly and heavily completely destroying Customer's expensive portable computer which except for Thief's actions would have been in the waterproof pocket of the coat and would not have been damaged. Does Customer have an action for trespass to chattel or conversion against Thief? A Customer has both an action for conversion and for trespass to chattel against thief. B Customer only has an action for trespass to chattel against thief but not an action for conversion. C Customer only has an action for conversion against thief but not trespass to chattel. D Customer has neither an action for trespass to chattel nor conversion against thief.

B Correct. Conversion requires substantial interference by control. As the restatement suggests, the ultimate question is whether the defendant "exercised so much control over the chattel that courts can justly require him to pay its full value." Many of the decisions indicate that the 3 hour time period of control would not be substantial enough to support conversion. The rule is oft stated that in order for trespass to chattel to be actionable the chattel actual harm to the chattel must occur. Here there is no actual harm to the chattel and at first blush the tort would therefore not seem actionable. However, when interference with the chattel causes harm which would not otherwise have occurred if the chattel was not interfered with then the tort is actionable. In this case even though the coal was actually harmed the plaintiff's expensive portable computer was destroyed as a result of the interference with the coat. Therefore trespass to chattel might be actionable but not conversion.

Driver was angry at corporate employee who he says cut him off the previous week in traffic. Driver followed employee to his place of employment and observed that employee parked in the same spot on the third floor of the same parking garage every morning. Every other spot was always occupied in the garage. Driver waited until the garage was empty of people but full of parked cars and placed a powerful explosive device under Driver's car. The device unsurprisingly to Driver was so powerful it caused the destruction of the employee's car but also caused the complete destruction of the five cars on either side of the employee's car to suffer damage. May the other people whose cars were damaged maintain a conversion action against driver? A No, because driver only had the specific intent to harm employee's car. B Yes, if driver was substantially certain the other people's cars would be completely destroyed. C Yes, if driver knew that it was reasonably foreseeable the other people's cars would have been. D No, even if the driver was substantially certain the other people's cars would have been destroyed.

B Correct. If the driver did not specifically intend harm to the other people's cars he might still be found to have had the requisite intent if he was substantially certain the harm would result. In these facts the driver knew every other spot was always occupied. The driver was not surprised that the explosive device he planted for the specific purpose of destroying employee's car was so powerful it also destroyed five cars on either side. So even though he did not specifically intend to damage the other people's cars if the facts are such that he was substantially certain the bomb was powerful enough to cause the destruction of these other cars then even if he was indifferent about their destruction he will be deemed to have intended their destruction as a matter of law.

In a negligence action, what is the plaintiff required to prove in order to establish proximate (or legal) causation? A The plaintiff is required to prove that but for the defendant's breach of the standard of care, the plaintiff would not have suffered the complained of harm. B The plaintiff is required to prove that the defendant should have reasonably foreseen, as a risk of her conduct, the general consequences or type of harm suffered by the plaintiff. C The plaintiff is required to prove that the defendant should have reasonably foreseen, as a risk of her conduct, the general consequences of type of harm suffered by the plaintiff and the manner in which the harm came about. D The plaintiff is required to prove that the defendant should have reasonably foreseen, as a risk of her conduct, the general consequences of type of harm suffered by the plaintiff, the extent of the harm, and the manner in which the harm came about.

B Correct. It is a correct statement of proximate or legal causation. The plaintiff is required to prove that the defendant should have reasonably foreseen, as a risk of her conduct, the general consequences or type of harm suffered by the plaintiff.

Twin 1 has an identical twin brother, Twin 2. Twin 1 is a violent person while Twin 2 is a docile, benevolent individual. Twin 1 and Twin 2 always dressed in identical fashion however. One day jogger was confronted by Twin 1 who told jogger, "If I ever run into you again I will kill you instantly." Jogger kept on jogging and as he turned the corner he saw Twin 2 standing on the side of the road. Twin 2 approached jogger quickly to inquire how jogger's day was going. Thinking Twin 2 was Twin 1, jogger began throwing large rocks at Twin 2 hitting him many times under the mistaken belief he had to defend himself as his own life was in danger. Jogger stopped throwing stones and ran away quickly as soon as Twin 2 stopped approaching him. In the lawsuit Twin 2 v. Jogger for battery, jogger claimed that he reasonably mistook Twin 2 for Twin 1 and acted as any reasonable person would have in the situation and defended himself. Which of the following legal arguments is correct? A Twin 2's argument that the reasonableness of jogger's actions is irrelevant in intentional tort law because intent is subjective B Jogger's argument that if a reasonable person would have done what he did he should be exempt from liability C Twin 2's argument that he never intended the jogger any harm and thus the jogger has no basis for applying the self-defense doctrine D Even if jogger continued to throw stones at Twin 2 after Twin 2 ceased approaching him he would not be liable because there is no limit to the level of force permitted in defending oneself.

B Correct. It is correct for a reason that confuses many tort students. There are two aspects to establishing liability in intentional tort. The first is that the plaintiff must establish all the elements of the tort exist. In this case the tort is battery. One element of the tort of battery and all intentional torts is intent. Whether the defendant possessed the requisite intent is absolutely a subjective standard. In other words intent is proven by showing that the particular defendant had the specific intent or was substantially certain. The knowledge of the reasonable person is not imputed as a matter of law onto the defendant in establishing the defendant's intent. In negligence however the defendant is deemed to possess all the knowledge of a reasonable person. In this context the proof of intent is said to be subjective (specific to the defendant) and the proof of negligence is said to be objective (not dependent on the defendant's particular knowledge etc). But once the plaintiff has established that all the elements of the tort have been met liability is not automatic. The defendant could admit that all elements of the tort have been met but if the defendant then proves the existence of a privilege the defendant is absolved from liability despite all the elements of the tort being met. One such privilege is self-defense. A person is privileged to use reasonable force to defend themselves. In contrast to the intent element reasonableness of the defendant's actions is the touchstone of whether or not a privilege can be asserted. Another rule is that if a person makes a reasonable mistake in exercising a privilege the privilege remains valid. Here, the facts state the twins were identical and dressed in the same clothes. Jogger reasonably mistook Twin 2 for Twin 1 and defended himself from a perceived death threat. Jogger's argument that if a reasonable person would have acted as he did he is exempt from liability is exactly right in terms of establishing a privilege.

About two years ago, Client was a patient of Ophthalmologist. Client alleges that during the time that he was Ophthalmologist's patient, Ophthalmologist improperly performed LASIK surgery on Client's eyes. The LASIK surgery was supposed to correct Client's poor vision. Instead, the surgery was performed skillfully, but resulted in blindness in both eyes, a risk about which Client was never informed and a reasonable person would not have known. If Client had known of the risk of blindness, he would not have opted for the surgery. Client hires Attorney Matthew to sue Ophthalmologist for medical malpractice. Attorney Matthew fails to file the complaint to initiate the cause of action against Ophthalmologist within the applicable statute of limitations. As a result, the court dismisses Client's case against Ophthalmologist. Client then hires Attorney Becky to represent him in a cause of action against Attorney Matthew for legal malpractice. In the legal malpractice action brought by Client against Attorney Matthew, what duty applies to Attorney Matthew? A Attorney Matthew had a duty to act as the average lawyer under the same or similar circumstances. B Attorney Matthew had a duty to act as the reasonable prudent lawyer under the same or similar circumstances. C Attorney Matthew had a duty to act as the reasonable prudent lawyer in the same or similar locality. D Attorney Matthew had a duty to act as the reasonable lawyer with Attorney Matthew's same training and experience.

B Correct. It provides the correct standard of care in a professional negligence action. An attorney has a duty to act as the reasonable prudent attorney in the same or similar circumstances.

A state administrative regulation prohibits railroad trains from blocking highway crossings for more than 10 minutes. Railway allows one of its trains to remain in a highway crossing for 30 minutes. Fifteen minutes into this 30-minute period, Driver, who is driving his car on the highway, fails to notice the train until it is too late and collides with it, suffering an injury. Driver sues Railway in negligence. At trial, Driver concedes that his negligence should reduce his recovery under comparative negligence principles, but claims that Railway is negligent per se for violating the regulation. The evidence at trial reveals that the history of the regulation and of the agency's findings accompanying the regulation show that the only purpose of the regulation is to encourage the free flow of traffic and prevent traffic delays. What effect, if any, does Railroad's violation of the regulation have on Railroad's liability for negligence? A Driver should be able to use evidence of the regulation's violation to establish that Railway's negligence because it is a state administrative regulation applicable to railroad trains. B Driver should not be permitted to use the regulation's violation to establish Railway's negligence because the prevention of personal injuries is not part of the regulation's purpose. C Driver may not use the regulation's violation as proof of Railway's negligence per se because Driver has conceded his own negligence. D Driver should not be permitted to use evidence of the regulation's violation to establish that Railway was negligent because administrative regulations may not be used to establish the standard of care in a negligence action.

B Correct. The facts explain that the regulation's only purpose was to encourage the free flow of traffic and prevent traffic delays. Therefore, it was not enacted to prevent the type of harm about which Driver is complaining, a car colliding with a train.

Practical joker familiar to shopper pulls out a small feather from his pocket and while laughing tells shopper, "I am going to tickle you with this feather now," and approaches. Shopper has told practical joker before not to do this exact same thing. Practical joker approaches shopper with feather slowly, in an unthreatening manner and begins to lightly tickle shopper on shopper's arm. Shopper warns practical joker to stop. When practical joker refuses to stop shopper shoots practical joker in the leg with a powerful handgun causing permanent injury. Assuming the tickling would be considered a battery, which of the following statements is correct? A Because shopper warned practical joker first and practical joker did not heed the warning shopper is entitled to use any force to stop the battery. B Shopper exceeded the scope of the privilege of self-defense. C Because practical joker's actions constitute a battery shopper was privileged to use any force to defend himself. D Practical joker as the instigator of the altercation is barred from recovering damages from shopper for injuries to practical joker's leg.

B Correct. The rule in tort law is that the use of force to defend oneself is limited to the force reasonably necessary in the situation. The amount of force reasonable in self-defense is directly related to the magnitude of the threat. The use of deadly force or force likely to cause grievous bodily harm is only permitted when the person asserting the privilege of self-defense reasonably believes they are being attacked with similar force. Here the tickling of an arm with a feather is not the equivalent of deadly force and so deadly force in self-defense is impermissible. The self-defense privilege is not a privilege which allows unlimited use of force in any situation. Rather the scope of the force permitted in exercising the privilege is directly related to the magnitude of the force used by the attacker. Shooting someone in the leg is beyond the scope of permissible force levels for being tickled on the arm.

Alice was exposed to two different cleaners while working in a laboratory. Each cleaner contained a toxic chemical. One contained Chemical B; the other contained Chemical C. After developing the dreadful disease decolis several years later, Alice sues the manufacturers of each solvent, claiming that the manufacturers were negligent for including a toxic chemical in their solvents. Alice's evidence, presented by competent expert testimony based on valid scientific evidence reveals that the increased risk of contracting decolis from the dose of Chemical B to which she was exposed is insufficient to permit a finding of factual causation. Similarly, the increased risk of decolis from exposure to Chemical C is insufficient to permit a finding of factual causation. However, Alice's evidence reveals that while Chemical B and Chemical C operate independently, the combined risk of contracting decolis due to exposure to both is sufficient to permit a finding of factual causation. Which manufacturer is the factual cause of Alice's decolis? A Neither manufacturer is a factual cause of Alice's decolis because neither was sufficient alone to cause the disease in Alice. B Each of the manufacturers is a factual cause of Alice's decolis because the combined exposure to both was sufficient to cause her to develop the disease. C Neither manufacturer is a factual cause of Alice's decolis because, but for Alice's exposure to both, she would have not become sick. D Neither manufacturer is a factual cause of Alice's decolis because she has failed to satisfy her burden of production on the issue of factual causation.

B Correct. The rule is that when there are two insufficient, independent causes, each is a factual cause.

Driver is driving his car at 60 miles per hour on a stretch of highway with a 50 mile per hour speed limit when he runs into Pedestrian, seriously injuring her. In her negligence action against Driver to recover for her injuries, Pedestrian alleges that Driver was negligent in exceeding the speed limit. In order to establish factual causation (also known as causation-in-fact) as part of her negligence action, what will Pedestrian need to prove? A Driver ran into her when he was speeding. B Driver would not have hit her or would have caused her less harm if he had not been exceeding the speed limit at the time she was hit. C It was foreseeable to Driver that while speeding, he would run into a person walking along the highway thereby causing physical injury. D It was foreseeable to Driver that while speeding he would run into Pedestrian, causing her injuries.

B Correct. This question asks about establishing factual causation (also known as factual causation or but for causation) in a negligence action. It is essential to a plaintiff's negligence action that she prove that the harm about which she is complaining would not have occurred absent the negligent conduct. Restatement (Third) Torts § 26.

Hospital negligently misidentifies a corpse, causing it to be cremated rather than sent to a funeral home for burial as directed by the family. As a result of Hospital's negligence, members of the family suffer serious emotional harm upon learning of the Hospital's mistake. In a lawsuit brought by the family against Hospital for its negligence in misidentifying their family member, may Hospital be subject to liability in negligence? A Hospital may not subject to liability in negligence because the family's injuries are purely emotional. B Hospital may be subject to liability in negligence because the family suffered severe emotional injury due to negligent mishandling of a corpse. C Hospital may not be subject to liability because its negligence resulted in injury to a corpse, which is not compensable in tort law. D Hospital may not be subject to liability in negligence because pure emotional distress is only compensable for the intentional infliction of emotional distress.

B Correct. Typically, pure emotional distress alone is not compensable in a negligence action. However, there are exceptions, including when a defendant's negligence causes serious emotional harm during the course of an undertaking especially likely to cause serious emotional harm. Mishandling of a corpse is one of those undertakings.

Employee worked as the maintenance manager for a 500 acre estate. The entire estate was fenced. One day the owner of the estate got angry at employee. He locked employee inside the estate and called employee on the telephone, "I am angry at you, and so you won't be allowed to leave the estate for a week." Employee was forced to stay in the estate for the week. During the week, and with the permission of the owner of the estate, the employee lounged in the fancy house on the estate and consumed the fancy food and drink. The employee's view was, "As long as I am forced to stay here I might as well make the best of it and enjoy the level of luxury that I am normally not privy to." In his suit for false imprisonment against the owner: A Employee will not prevail because the grounds were more luxurious than he was used to B Employee will prevail even though the grounds were more luxurious than he was used to C Employee will not prevail because 500 acres is too large an area to be considered imprisonment D Employee will prevail because the confinement was for more than a short time

B False imprisonment is the intentional confinement of the plaintiff within identifiable boundaries. Confinement implies that the plaintiff is held against his or her will. Here, the luxury aspect of it might reduce the harm suffered by the employee and hence the damages compared to if he was confined, for example, in a filthy small cell with no food and water. But false imprisonment exists simply because he was restrained against his will. Unless there is authority of justification or another privilege existing, there is a right to be free from confinement. In other words, false imprisonment is a dignitary tort and it is the impermissible restraint of the employee in this case which is the basis of the action

Homeowner hears on the news that there is a dangerous criminal on the loose in her neighborhood. She is on edge as a result and hyper vigilant. One evening at dusk she is taking out her trash to the street and is startled by her neighbor who walked up behind her to give her some pie. Thinking it was the criminal approaching her she swings a large piece of wood at the neighbor causing him a pretty painful broken arm. Neighbor sues homeowner for battery. Homeowner argues that even though she committed a battery there should be no liability because she reasonably mistook the neighbor for the dangerous criminal and she defended herself accordingly. Will Homeowner's defense succeed? A) No, because she made a mistake. B) Yes, if factfinder concludes she made a reasonable mistake. C) No, because she used force in defending herself. D) Yes, whether or not her mistake was reasonable.

B. Here the homeowner is not arguing mistake negates intent, but she is arguing that she had the right to defend herself or to exercise the privilege of self-defense. While it is true that mistake does not negate intent, the commission of a reasonable mistake in the exercise of a privilege does not negate the privilege. In contrast to the previous question homeowner is not seeking to make the legally impossible argument that mistake negates intent, rather she is arguing that she was privileged to defend herself. B is correct because reasonable mistake does not negate the privilege. So even though homeowner committed a battery there is no liability because of the privilege of self-defense.

Passenger was angry at taxi driver because passenger thought he was charged too much. They both began arguing outside the car at passenger's destination. In a fit of anger passenger attempted to kick taxi driver. Taxi driver blocked the blow by putting his tablet computer in the path of passenger's foot. Although taxi driver was unharmed his tablet computer was destroyed in the process. If taxi driver sues passenger for trespass to chattel what is the likely result? A Taxi driver will lose because passenger did not have the intent to interfere with his chattel. B Taxi driver will prevail even though passenger never intended to harm his chattel. C Taxi driver will lose because the chattel was destroyed and so the only appropriate cause of action is conversion. D Taxi driver will prevail because intent is not required for trespass to chattel or conversion.

B. The doctrine of transferred intent applies to all the torts within the original writ of trespass. As a reminder these torts are Trespass to Chattel, Trespass to Land, False Imprisonment, Assault and Battery (TTFAB). The facts clearly indicate that the passenger attempted to commit a battery on the taxi driver. What happened however, was the destruction of the tablet computer. Because a tablet computer is personal as opposed to real property it is considered a chattel. As a result the intent from the battery attempt will transfer so that trespass to chattel will be satisfied. Trespass to chattel is intentionally interfering with the plaintiff's possession of a chattel in a way that causes recognizable harm. Based on that definition the only element of the tort which is unsatisfied is the intentional aspect. However the intent prong is satisfied because of transferred intent and as a result trespass to chattel is actionable.

Hiker was hiking in the woods. As hiker approached a bend in the path a large tree branch snapped, hitting hiker and sending her airborne for about 30 yards. She landed relatively unhurt but in the process was thrown over the unmarked property line of landowner, landing on landowner's land damaging some shrubs. If landowner sues for trespass to land, will landowner win or lose? A) Landowner will win because hiker tangibly entered onto her property. B) Hiker will win because her entry onto landowner's property was not volitional. C) Landowner will lose because the price of shrubs pales in comparison to human life. D) Hiker will lose because her entry onto the land caused actual damage.

B. The entry onto the land was non-volitional. Intent requires a volitional or voluntary act to bring about a tortious result. Here there was no volitional act on the part of the hiker to enter the landowner's property.

Tonya lay in wait for Michael. She hated him and her hate made her certifiably insane and this time she was going to get him. As Michael cruised along the highway Tonya hid in the bushes at the side of the road and planned to ram a huge piece of metal into the rear wheel of Michael's motor bike. The plan was a sudden stop, causing Michael to fly over the front of the bike and to get severely cut as he dragged along the road. As Tonya rammed the wood into the wheel Michael screamed in terror and then flew over the bike and got severely injured from his contact with the road and suffered a severely disabling emotional response. His bike was also completely destroyed in the process. Tonya did not intend to scare him or to cause severe emotional harm or to wreck his bike. What causes of action does Michael likely have? A) Michael likely has a cause of action for battery only. B) Michael likely has a cause of action for assault and battery. C) Michael likely has a cause of action for assault, battery and intentional infliction of emotional distress. D) Michael likely has a cause of action for battery, assault, intentional infliction of emotional distress and conversion.

B. Tonya clearly intended to cause a battery, but also ended up scaring Michael as evidenced by his "scream in terror." Therefore because of the doctrine of transferred intent Tonya's intent to commit battery will also result in an assault because fear or apprehension of harmful or offensive contact also resulted.

As bar owner was closing up for the night adult drunk patron walked into the bar and asked for a drink. Bar owner apologized and said that he was closing up and would not be serving anyone else. Drunk patron then told Bar Owner that if he would serve him three more drinks, Bar Owner could have his antique car which was parked in the parking lot of the bar. Bar owner walked outside looked at the car and agreed. After the third drink Bar Owner led drunk patron outside of the bar, locked up and then Bar owner drove away in drunk patron's car. If bar tender was reasonably unaware that bar patron was drunk during this period, what result if drunk patron subsequently sues Bar owner for conversion? A) Drunk patron will prevail if he was so drunk he lacked the capacity to actually consent to Bar owner taking his car. B) Drunk patron will not prevail even if he was so drunk he lacked the capacity to consent to Bar owner taking his car. C) Bar owner will prevail even if Drunk patron was so drunk he lacked the capacity to consent because incapacity does not negate consent. D) Bar owner will prevail even if Drunk patron was so drunk he lacked the capacity to consent because only the incapacity of minority can negate consent.

B. When a person lacks the capacity to give consent and the person to whom the consent is given should reasonably be aware of the incapacity there is not valid consent. Here however the person to whom consent was given was reasonably unaware of the Drunk patron's incapacity. When the person to whom consent is given is reasonably unaware that the consenting person did not have the capacity to consent the consent is valid even though the consenting person actually lacked the capacity to consent.

Shopper insulted store clerk by saying that the store clerk had a disgusting body odor. Shopper intended to insult and embarrass store clerk and said it loud enough for the fellow shoppers in the vicinity to hear. Store clerk was mortified when other shoppers laughed. As a result of the insult, store clerk fainted and hit his head on a display and suffered a concussion. If store clerk brought an action for Intentional Infliction of Emotional Distress against shopper, would store clerk recover? A Store clerk would recover because he intended to cause emotional harm and did in fact cause emotional harm. B Store clerk would likely not recover because the shoppers comment is legally insufficient to support a cause of action for IIED. C Store clerk would recover if the harm suffered was indeed severe. D Store clerk would not recover if he is considered the equivalent of an innkeeper.

B. Intentional infliction of emotional distress requires more than an act intended to cause severe emotional distress and the resulting distress. In order to be actionable, not only must the tortfeasor intend severe emotional harm but his actions which bring about the severe harm must also be "extreme and outrageous." The restatement has defined this standard as acts which are beyond all bounds tolerated by a civilized society. Mere insults have been held not to fall within the category of extreme and outrageous. Therefore even though the shopper intended to cause emotional harm and acted deliberately to achieve that purpose and did in fact achieve the intended result the insult is legally insufficient to sustain the action.

A martial arts expert with incredibly fast reflexes and catlike agility was walking along the street when a stranger approached her. Stranger knew of Martial Arts Expert's reputation for speed and wanted to see how fast Martial Arts Expert really was. Stranger was 3 feet in front of Martial arts expert when he attempted to strike her shoulder with his hand. Even though it was moving quickly, Martial Arts Expert saw the stranger's hand and arm moving as if it was in slow motion. In her mind she was calm and experienced no fear whatsoever. Not only was the stranger laughably slow but she would not have gotten hurt if the strike made contact with her. She merely thought, "I'm just going to move out of the way of his hand because I don't want any strangers touching me." She moved out of the way of the strike and stranger fell. Is stranger liable for assault? A No, because stranger did not attempt or intend to cause assault but only attempted to cause a battery so the element of intent is missing B No, because Martial Arts Expert was never fearful of being hit C Yes, because Martial Arts Expert was aware of an imminent unconsented to contact which would be harmful or offensive to a reasonable person D Yes, because the stranger could actually have reached the Martial Arts Expert

C Correct. Assault does not require fear but fear OR Apprehension and Apprehension is defined as the mere anticipation or awareness of an unconsented to touching. Remember, unconsented to touching qualifies as offensive if we remember that battery is a dignitary tort. So, martial arts expert's awareness that an unconsented, offensive touching would occur unless she took evasive action is enough for apprehension to be established. Here, the facts say that the stranger's hand was moving quickly so even though it appeared to be moving slowly to martial arts expert, the contact was imminent.

Drug Manufacturers (a total of seven companies) manufactured, produced, and marketed a medication, DRG, designed to prevent miscarriages in pregnancy. The drug was administered to pregnant women. Unfortunately, women who ingested DRG, thereby exposing their unborn daughters to DRG, gave birth to daughters who developed cancerous vaginal and cervical growths. The unique type of cancer these daughters developed was a fast-spreading and deadly disease requiring radical surgery to prevent its spread. Daughter's mother ingested DRG, exposing Daughter to the drug prior to her birth. The evidence also shows that this unique type of cancer is most probably caused by exposure to DRG. Daughter has developed the cancerous vaginal and cervical growths, which are only developed by women like her whose moms ingested DRG. Daughter has sued Drug Manufacturers, all of whom manufactured, produced, and marketed DRG at the time that Daughter's mother ingested the drug. Drug Manufacturers represent 90% of the market share of manufacturers of DRG. Daughter seeks to hold them responsible in negligence for their manufacture of DRG and Daughter's resultant injuries. However, Daughter is unable to prove with certainty which one of the seven defendants actually manufactured the DRG that her mother ingested. Drug Manufacturers have moved for summary judgment, arguing that Daughter's cause of action against them should be dismissed for failing to establish factual causation. In a jurisdiction that recognizes market share liability, should the court grant defendants' motion for summary judgment? A Yes, evidence that Drug Manufacturers produced the drug to which Daughter was exposed is insufficient to establish factual case. B No, Drug Manufacturers produced the drug at the time that Plaintiff was exposed to DRG in utero and therefore are responsible for her injuries. C No, as long as Defendants represent a substantial share of the market of those manufacturing DRG and at the time Daughter's mother ingested DRG, each defendant may be responsible. D Yes, defendants cannot be subject to liability for injuries caused by a drug to which Plaintiff was exposed prior to her birth.

C Correct. For an example of a market share liability jurisdiction, see Sindell v. Abbott Laboratories, 26 Cal.3d 588 (1980), in which the Supreme Court of California permitted a lawsuit to proceed against a manufacturer of a drug in the absence of proof of factual causation when the plaintiff had brought before the court defendants representing 90% of the market share.

As part of a game of truth or dare Melissa walked toward a perfect stranger, and when she was about 10 feet from the stranger she asked the stranger the time. As the stranger looked up from his watch, Melissa removed an unloaded gun from her pocket, pointed it at the stranger, and pulled the trigger. As he saw this, the stranger yelled in terror, jumped back, and raised his hands. Melissa laughed uncontrollably at the stranger's reaction and then told him it was a simple prank and the gun was unloaded. Will Melissa be liable for assault and battery? A Melissa will not be liable for assault because the gun was not loaded. B Melissa will not be liable for either battery or assault because she was not close enough to touch the stranger when she pulled the gun. C Melissa will be liable for assault even though she was not attempting to commit a battery. D Melissa will be liable for battery if she is liable for assault because assault can never occur independently of battery.

C Correct. In assault, it is the apparent ability to effectuate harmful or offensive contact rather than the actual ability to cause contact that is legally sufficient. For example, here, even though the gun was not loaded, and as a result there was no actual ability to effectuate the harmful or offensive contact ,it reasonably appeared to the stranger (apparent ability) that harmful or offensive contact was imminent. If the victim of the tort reasonably believed harmful or offensive contact was imminent, then it is irrelevant to the tort whether there was actually the ability to bring about the contact. The tricky part of this question is that many students erroneously assume that no assault is possible unless there is a battery. That is incorrect. Assault is an independent tort which does not depend on the existence of a battery and vice versa.

Jessica was driving her car, which was manufactured by Motor Company, when the voltage regulator in the car failed due to its negligent installation. The failure caused the battery fluid to boil, which produced toxic fumes that reached the interior of the car. Jessica suffered chronic vocal-cord damage as a result. Prior to this occurrence, Jessica was a popular singer who earned several millions of dollars each year. Jessica sues Motor Company for its negligence to recover for her injuries. Will Jessica be able to recover for her injuries? A Jessica will only be able to recover for injuries that the ordinary person would have suffered due to the same occurrence, as Jessica being a popular singer was unforeseeable to Motor Company. B Motor Company is only subject to liability for damages to the vehicle itself, not for injuries to a person. C All of Jessica's lost earnings due to her vocal-cord injury are within the scope of Motor Company's liability because the type of injuries Jessica suffered were foreseeable to Motor Company from its negligence. D None of Jessica's lost earnings due to her vocal-cord injury are within the scope of Motor Company's liability because those injuries were not foreseeable to Motor Company from its negligence.

C Correct. Jessica's lost earnings due to her vocal-cord injury are within the scope of Motor Company's liability were foreseeable to Motor Company from its negligence. This is an application of the eggshell skull plaintiff rule.

Visitor was a customer of Barge, when while on the deck of Barge he accidentally fell into the water as the barge approached the pier. Employee of the barge hears Visitor's cries as he yells from the water that he cannot swim. Employee threw a heavy line (used to tie the barge to the pier) in the direction of the cries. The line came within two feet of the spot of Visitor, but Visitor did not grab the lines. Visitor was carried away from the pier by the outgoing tide and downed. At the time of this incident, the employee was aware that several life preservers were on a rack located near the front of the barge where Visitor fell overboard. The preservers remained in the rack and were not used. If the preservers had been used, Victim probably would not have drowned. Visitor's estate sues Barge in negligence, alleging that the failure to use the life preservers was the factual cause of Victim's drowning. Is Visitor's estate correct that Barge's failure to use the life preservers was a factual cause of Victim's drowning? A No because the failure to use the life preservers is an omission and an omission cannot be the basis for a negligence action. B No because it is possible that Victim would have drowned even if the life preservers had been thrown out to him. C Yes because not throwing the preservers to Victim was a substantial factor in bringing about Victim drowning. D Yes because Victim drowned when he was a customer of Barge.

C Correct. Negligence (breach of the standard of care) is a factual cause of a plaintiff's injury if it is more likely than not that the harm would have been averted but for the negligence of the defendant.

A major fire is approaching city. Between city and fire are five houses. These five houses will act as a bridge to communicate the fire into the city if they ignite. As a result the mayor of city orders the 5 houses destroyed despite the protests of the homeowners before the fire reaches them. The destruction of the houses prevents the destruction of the entire city. Homeowners sue the mayor and the city for trespass to land. Will homeowners prevail? A Homeowners will prevail because their private property was destroyed without consent. B Homeowners will prevail but liability would only exist for actual harm because the mayor and city were acting under the doctrine of necessity. C Homeowners will not prevail and there will be no liability because of the doctrine of public necessity. D Homeowners will not prevail because it was a government official who made the decision that necessity warranted destruction of their houses.

C Correct. Public necessity is defined as follows, "A defendant who damages, destroys, or uses the plaintiff's property in the reasonable belief that by doing so he can avoid or minimize serious and immediate harm to the public is protected against liability for intentional torts by a complete privilege." In contrast to private necessity the doctrine of public necessity is a complete privilege and the defendant if properly acting under the privilege is completely free of liability, even liability for actual damage.

As Plaintiff is walking down the street, a chair falls out of Warehouse's window, striking Plaintiff on the head. Plaintiff seeks to sue Warehouse in negligence and wants to employ the doctrine of res ipsa loquitur to do so. When using the doctrine of res ipsa loquitur to prove a Warehouse's negligence, Plaintiff: A most likely has additional direct evidence of the defendant's negligence. B most likely has additional circumstantial evidence of the defendant's negligence. C must prove that the accident does not ordinarily occur in the absence of negligence. D must specifically identify what the defendant's negligent act entailed.

C Correct. Res ipsa loquitur requires proof that the accident is the type of accident that does not ordinarily occur in the absence of negligence. Restatement (Third) Torts § 17.

Bully intends to cause victim to suffer severe emotional distress. As a result Bully wants to make victim look weak in front of other classmates. As victim is approaching his locker between classes Bully sneaks up behind victim and gently says, "Hey." As victim turns around Bully screams right in his face causing victim to lurch backwards out of fear and surprise and to hit the back of his head on the locker from fear of what the victim thought was an imminent physical attack on him. Assume the prank by the bully is not considered extreme and outrageous and the bully never intended for the victim to suffer physical harm. How may the victim recover for the injuries suffered when he hit his head? A Sue for battery because Bully intended extreme emotional distress and even though that tort is not actionable the intent form the emotional distress tort will transfer rendering battery actionable. B Sue for battery because even though Bully did not intend physical harm it would have been a foreseeable result to a reasonable person. C Sue for assault and battery because the bully was probably substantially certain that his actions would cause apprehension of harmful or offensive contact and the intent from assault will transfer to battery rendering the tort of battery complete. D Sue for trespass to chattel because the victim's head hit the locker causing damage to the locker and the intent for trespass to chattels will transfer to make assault and battery actionable.

C Correct. The bully likely intended to scare the victim if he yelled in his face. Even if his ultimate specific intent was to embarrass the victim in front of his peers the Bully likely knew or at the very least was substantially certain that the victim would be fearful or apprehensive of contact after he screamed. Once this substantial certainty is established for assault, intent is established for all the five torts within the original writ of trespass, (Trespass to Land, Trespass to Chattel, False Imprisonment, Assault and Battery). Therefore even if the bully never intended harmful or offensive contact the intent from assault would transfer satisfying the intent requirement of battery and because harmful or offensive contact occurred, battery would be actionable.

In a lawsuit brought by Plaintiff against Defendant alleging negligence, Plaintiff seeks to use Defendant's violation of a statute to help establish his prima facie case. The court conducts an inquiry to determine: (1) whether Plaintiff is a member of the class the legislature intended to protect in enacting the statute; (2) whether the hazard about which Plaintiff is complaining is one the legislature intended to prevent; and (3) whether tort liability is appropriate. How is conducting this three step analysis important to Plaintiff's claim against Defendant? A It establishes whether Plaintiff may use the doctrine of res ipsa loquitur. B It establishes the effect of proof of the statute's violation. C It establishes whether the statute's violation can be used to establish negligence (also known as negligence per se). D It establishes whether a Rule of Law may be used to establish negligence.

C Correct. The determination is one of whether a statute can be used to establish a defendant's breach of the standard of care (i.e., negligence).

Insane person perceives caregiver as a threat even though caregiver is not a threat. As a result insane person violently attacks caregiver and confines caregiver in a restroom for 3 days. Caregiver sues insane person for battery and false imprisonment. Will insane person be liable? A Insane person will not be liable because insane people cannot have intent. B Insane person will be liable unless he acted like the reasonable insane person under the circumstances. C Insane person will be liable because insanity does not negate intent. D Insane person will not be liable if he made a mistake in defending himself.

C Correct. The general rule of law in tort is that insanity does not negate intent. As long as the insane person is capable of intent, for example in this case, intending to hit the caregiver who the insane person erroneously perceived as a threat, the insanity is irrelevant in establishing intent.

Farmer is a country farmer who has a lower IQ than most individuals. However, he is also unusually physically fit, much physically bigger and stronger than the average person. Lately, as part of his trade as a farmer, he has been stacking hay on his property. However, he has been doing so in a way that a farmer of ordinary intelligence would know will cause the hay to combust spontaneously into flames. Defendant's hay does in fact combust causing a fire, which spreads to Neighbor's property causing property damage. In the negligence action brought by Neighbor against Farmer, how should the court instruct the jury regarding the reasonable prudent person standard of care? A The reasonable prudent person standard will take into account that Farmer has a lower IQ than most individuals. B The reasonable prudent person standard will not take into account that Farmer was physically bigger and stronger than the average person. C The reasonable prudent person standard will not take into account that Farmer has a lower IQ than most individuals. D The reasonable prudent person standard is subjective and based on Farmer's qualities.

C Correct. The reasonable prudent person standard does not take into account mental incapacities of the defendant.

Art owns a convenience store located within an exclusive, gated community. Sarah, who lives in the community, tripped on a curb in the store's parking lot one night after emerging from the store, suffering a skull fracture in the fall. Sarah sues Art for negligence in failing to have lights illuminating the parking lot, which without the lights is pitch dark. If the factfinder determines that Arthur's failure to light the parking lot was negligent and a factual cause of Sarah's harm, which of the following statements about causation is correct? A Sarah has established the causation element of the negligence action against Arthur. B The factfinder is likely to determine that Art's negligence is not a proximate cause of Sarah's harm because the risk of someone falling in the parking lot due to darkness is not among the risks that made Art negligent for failing to provide lighting. C The factfinder is likely to determine that Art is a proximate cause of Sarah's harm as well because the risk of someone falling in the parking lot due to darkness is among the risks that made Art negligent for failing to provide lighting. D The court must grant judgment in favor of Sarah.

C Correct. This question asks about proximate causation determinations once the factfinder has established that the defendant's negligence was a factual cause of plaintiff's injury. The statement that the factfinder is likely to determine that Art is the proximate cause of Sarah's harm as well because the risk of someone falling in the parking lot due to darkness is among the risks that made Art negligent for failing to provide lighting.

Apple County maintained a nature park that included paths along its canyons and gorges. Park Ranger, employed by the county, became aware that a stream had dangerously eroded the support for a path, but neglected to close the path or to post warnings. A group of campers pitched camp after dark near the path. One of the campers, Charles, left the group to go on a walk. Charles fell into a gorge when the weakness in the support for the path gave way. The other members of the camping party heard what sounded like a falling object and called out to Charles, but received no response. Sean proceeded with a flashlight to investigate and discovered that Charles had fallen into the bottom of the gorge and appeared to be unconscious. In his attempt to descend to the bottom of the gorge, Sean lost his footing, fell, and suffered harm. Sean sues Apple County and Park Ranger in negligence for his injuries. Who, if anyone, may be liable to Sean for his injuries? A Neither Apple County nor Park Ranger will be liable to Sean for his injuries because he chose to help Charles. B Neither Apple County nor Park Ranger may be held liable to Sean for his injuries because Charles's decision to go on a walk was not foreseeable to Apple County or Park Ranger. C Both Apple County and Park Ranger may be liable to Sean for his injuries because his rescue was a foreseeable consequence of their negligence. D Any cause of action Sean chooses to bring must be brought solely against Charles because Charles was in need of rescue.

C Correct. When a person's tortious conduct imperils another the actor's liability generally includes injury suffered by the rescuer in effectuating a reasonable rescue. Restatement (Third) Torts § 32. This question asks about the ability of rescuers to recover from an original tortfeasor. In the famous words of Judge Cardozo, "danger invites rescue."

Plaintiff is injured when Defendant briefly loses control of the car he was driving when he reaches for his cell phone he dropped on the floorboard and hits Plaintiff who was standing on the street corner. Witness was a passenger in the Defendant's car at the time of the accident and saw everything that happened. In the negligence action brought by Plaintiff against Defendant to recover for her injuries, Witness testifies that she was riding in Defendant's car while Defendant was driving, that she heard Defendant's cell ring, that Defendant went to answer his cell phone while he was driving but dropped it on the floorboard of the car, and that when Defendant went to pick up his cell phone while he was driving, he took his eyes off the road for a brief period of time during which he lost control of the car and struck Plaintiff. What is the proper characterization of the evidence presented at Plaintiff's trial against Defendant for Defendant's alleged negligence? A Witness's testimony is circumstantial evidence of Defendant's conduct that may be used to prove that Defendant breached the standard of care. B Witness's testimony is circumstantial evidence of Defendant's conduct that may not be used to prove that Defendant breached the standard of care. C Witness's testimony is direct evidence of Defendant's negligence and does not require the factfinder to draw an inference regarding Defendant's breach of the standard of care. D Witness's testimony is direct evidence of Defendant's negligence and requires the factfinder to draw an inference regarding the Defendant's breach of the standard of care.

C Correct. Witness's testimony of Defendant's unreasonable conduct while driving is direct evidence of Defendant's negligence (breach of the standard of care).

Homeowner hears on the news that there is a dangerous criminal on the loose in her neighborhood. She is on edge as a result and hyper vigilant. One evening at dusk she is taking out her trash to the street and is startled by her neighbor who walked up behind her to give her some pie. Thinking it was the criminal approaching her she swings a large piece of wood at the neighbor causing him a pretty painful broken arm. Neighbor sues homeowner for battery. Homeowner argues that the intent element of the tort is missing because she never intended to harm the neighbor but thought it was a criminal. Will Homeowner's defense be successful? A Yes, because mistake negates intent. B Yes, but only if she can prove her mistake was reasonable. C No, because mistake does not negate intent. D Yes, if she did not use excessive force when swinging the large piece of wood.

C It states the correct rule of law. Mistake does not negate intent.

Customer left her coat on a coat rack at the entrance to a restaurant. Thief decided to steal the coat and keep it for himself. After 5 months the thief had a change of heart and figured out where customer lived. Thief returned the coat to customer's address unharmed. Does Customer have an action for trespass to chattel or conversion against Thief? A) Customer has both an action for conversion and for trespass to chattel against thief. B) Customer only has an action for trespass to chattel against thief but not an action for conversion. C) Customer only has an action for conversion against thief but not trespass to chattel. D) Customer has neither an action for trespass to chattel nor conversion against thief.

C Trespass to chattel is intentionally interfering with the plaintiff's possession in a way that causes recognizable harm. Conversion is intentionally exercising dominion or control over another's chattel in a way that in inconsistent with the other's rights and deprives the other of the chattel. While the length of time of the deprivation is relevant in determining whether interference with a chattel is a trespass to chattel or if it is substantial enough to be conversion 5 months would typically be enough to be considered dominion or control for conversion. One clear distinction between conversion and trespass to chattels is that the former is actionable even if the chattel is unharmed but the latter requires actual harm to the chattel in order to be actionable. Here the chattel was returned "unharmed" and because there was no actual harm to the chattel there can be no action for trespass to chattel. In contrast 5 months is likely to be considered dominion and control over the chattel and even in the absence of actual harm Customer could demand that Thief keep the chattel and pay Customer the market value of the chattel instead. This would be a conversion.

Tourist was walking along public road in a town he was visiting. Tourist approached a fenced parcel of land. As tourist approached the fenced area he saw many signs that read, "Private Property. KEEP OUT." Believing no one was looking tourist jumped the fence to examine a flower he saw on the land. As soon as tourist landed on the other side of the fence landowner yelled at tourist and tourist immediately jumped back over the fence onto the public roadway causing no harm to landowner's property. If landowner sues tourist for trespass to land will landowner win or lose? A) Landowner will win because tourist knew that the land was owned by the landowner. B) Landowner will lose because tourist did not spend enough time on landowner's land to be considered a trespass to land. C) Landowner will win because tourist intentionally entered onto land owner's land. D) Landowner will lose because tourist did not harm landowner's land.

C Trespass to land is intentionally entering or causing direct and tangible entry onto the land in the possession of another. The only intent required is the intent to enter onto land. The trespasser does not have to know that the land is in the possession of another. All that is required is intentional entry onto land and the land you enter is in another's possession. The required intent is not the intent to trespass just the intent to enter land. Here the trespasser intended to enter land in the possession of another and therefore there was trespass to land.

Law student is angry at Torts professor because professor made student work and professor is always so prepared in class. As professor is walking to his car in his perfectly matching designer shoes the student confronts him and screams, "I am going to hit you with this book for being so prepared." Student did not intend to hit him but just to scare him by swinging the book close to his head and making him flinch. If everything went exactly as planned are the student's actions assault and/or battery? A) The students actions are both an assault and battery. B) The students actions are neither an assault nor a battery. C) The students actions are an assault but not a battery. D) The students actions are a battery but not an assault

C While there is no battery because battery requires contact. There is an assault. Assault is the intentionally causing fear or apprehension of imminent harmful or offensive contact. Here the student intended to scare the professor and the professor's flinch indicates he was at fearful or apprehensive of what he perceived to be imminent contact.

Two college students, A and B, were out drinking heavily one night. A third college student, C, who knew the drunk students watched as A and B passed out. C drove B to B's house and left B there. After doing this C thought it would be a great idea to drop the unconscious A on some random stranger's lawn. C pulled up next to an unfenced yard and dragged A onto the lawn. The next morning the irate property owner woke A up and asked A to leave. Did A and/or C trespass? A) Neither A nor C trespassed B) Only A trespassed C) Only C trespassed D) Both A and C trespassed

C. A could not have trespassed because trespass requires an intentional act to enter or cause tangible entry onto the property of another. A was unconscious and incapable of acting voluntarily therefore A was incapable of intent and cannot have trespassed. C however acted intentionally to cause the impermissible entry of A onto the land of another and therefore C trespassed. It is important to note that a person can be liable for trespass if they cause tangible entry and not only if they actually enter land. For example if a person X throws a tangible object over a person Y's fence into their yard, person X may be liable for trespass even though person X did not actually enter the land of person Y, This is because person X caused tangible entry onto the land of person Y.

Connie is carelessly changing lanes in her car when her car is struck by Bill who is reaching for his phone, which has fallen under the passenger side seat while he was driving. Adrian, who is in the vicinity on her motorcycle waiting patiently at the intersection for the light to turn green, is struck by Connie's car and suffers serious physical injury as well as damage to her bike. She sues both Connie and Bill for their negligence. The jury determines that Adrian's damages are $100,000 and concludes that Bill and Connie are both liable. It assigns 30% responsibility to Bill, and 70% responsibility to Connie. In a jurisdiction that subjects defendants to joint and several liability, if Adrian recovers $100,000 from Bill alone, what amount is Bill entitled to collect from Connie? A Bill is entitled to collect $100,000 in contribution from Connie. B Bill is entitled to collect $100,000 in satisfaction from Connie. C Bill is entitled to collect $70,000 in contribution from Connie. D Bill is entitled to collect $70,000 in satisfaction from Connie.

C. Correct. A tortfeasor who pays more than its fair share is entitled to contribution from other jointly and severally liable tortfeasors. Therefore, Bill is entitled to collect 70% of the $100,000 from the tortfeasor determined to be 70% at fault, Connie.

Toddler, a two-year-old child suffering from a bacterial infection, received a routine childhood vaccination with a vaccine manufactured by Vaco. Shortly after receiving the vaccine, Toddler spiked a very high fever, went into respiratory arrest, and died. Toddler's estate sues Vaco in negligence. At trial, Toddler's estate provides competent evidence that Toddler died due to a combination of the infection and the vaccine, and if either toddler had not had an infection or if Toddler had not been vaccinated, death would not have ensued. In its defense, Vaco presents evidence that the infection and its drug do not react together, but rather are entirely independent. Therefore, Toddler's death could only have been caused by the infection or the drug, but not a combination of both. The factfinder is persuaded by the defense's evidence and finds that the infection more likely than not caused the death of Toddler rather than the vaccine. Which of the following statements regarding the factual causation element of the negligence action is correct? A Toddler's estate has not provided evidence on the issue of factual causation. B Toddler's estate has satisfied its burden of persuasion that the vaccine was a factual cause of Toddler's death. C Toddler's estate has failed to satisfy its burden of persuasion that the vaccine was a factual cause of Toddler's death. D Vaco may be held liable for the death of Toddler even without proof of factual causation.

C. Correct. The factfinder concluded that the infection more likely than not caused the death of Toddler rather than the vaccine. Toddler's estate put on evidence from which the factfinder could have concluded that the death was caused by the vaccine, so Toddler's estate satisfied its burden of production. This question is more about the burden of persuasion rather than the burden of production. Had Toddler's estate satisfied its burden of persuasion, the factfinder would have found that the vaccine more likely than not caused the death of Toddler, but the factfinder did not.

Patron was trying to enter a large exclusive hotel in the middle of a city. The representatives of the hotel met patron at the door and casually informed him that he was not permitted to be on the hotel property. After that, they forcibly excluded him from the hotel property every time he tried to enter. If patron files a false imprisonment claim against the hotel what is the hotel's best defense? A False imprisonment can never occur on private property. B False imprisonment cannot occur because property peaceably informed patron he would be a trespasser the first time patron tried to enter. C Exclusion is never false imprisonment. D The remainder of the city other than the hotel was too large to be considered false imprisonment.

C; Correct. It restates an established rule of law. That rule is that exclusion is never false imprisonment. Here, patron was excluded from entering, and exclusion is almost the opposite of confinement.

Thief knowing that child was only 8 years old and home alone knocks on the door of child's house. Child opens door and thief befriends child. After a while thief tells child that he will buy child candy if child allows thief to come in and to take away anything thief wants. Child, excited about the prospect of candy, allows thief in. Thief takes away many chattels from the house and gives child candy as promised. Was the child's consent valid? A Yes. The child's consent was validly obtained because thief bought the candy as promised. B No. The consent was invalid because only the owner of a dwelling can give permission to enter. C Yes. The child's consent was validly given because the thief did not place the child in duress in order to obtain consent. D No. The child is incapable of giving consent.

D Correct. As a minor the child is incapable of giving consent.

School District provides school crossing guards at the three most dangerous intersections for each school in the district. While walking to school and crossing one of those intersections, Student, a seven-year-old, is hit by a car driven by Driver and injured because no crossing guard is present. Student's guardian sues School District, claiming that its negligence caused Student's harm. May School District be held liable in negligence? A School District may not be held liable in failing to provide crossing guards because it cannot be held liable in negligence law for its failure to act. B School District may be liable for Student's injuries because school districts are responsible for the well-being of their students when students are on the way to school. C School District may not be held liable because Driver is the only party who can be subject to liability for Student's injuries. D School District may be held liable for its negligence in not having a school crossing guard at the intersection where Student was injured because School District undertook the responsibility to provide crossing guards and had a duty to use reasonable care in doing so.

D Correct. School District, having undertaken to render a service, should know that its services reduce the risk of injury to another, and another relies upon the defendant to exercise reasonable care in doing so. See Restatement (Third) Torts § 42 and illustration 5 upon which this fact pattern is based.

Lincoln's airplane was seriously damaged when he was forced to land it without its retractable gear in the down position. Lincoln sues Dow Aircraft, the manufacturer of the plane. Lincoln claims that Dow Aircraft neglected to include instructions in its service manual of the need to be sure of a minimum clearance between two parts in the landing gear assembly when reassembling the gear after routine servicing. Lincoln's plane was serviced at Dow because Dow needed to complete unrelated warranty work to the aircraft. The Dow mechanic who worked on Lincoln's plane was fired shortly after the work was completed for repeatedly failing to consult service manuals. In a cause of action against Dow Aircraft and the mechanic, Lincoln includes a claim based on the mechanic's negligence. Which of the following statements regarding factual cause of the harm to Lincoln's plane is correct? A The omitted instruction by itself was a factual cause of the harm to Lincoln's plane. B The mechanic's negligence in failing to consult the manual was by itself a factual cause of the harm to Lincoln's plane. C Because neither the missing instructions nor the mechanic's failure to consult the manual were a factual cause of Lincoln's harm, Lincoln will not be able to establish that Dow Aircraft was a factual cause of the damage to his plane. D If Lincoln can prove both that Dow Aircraft providing the omitted instructions and a mechanic properly consulting the service manual would have prevented the damage to Lincoln's plane, Dow Aircraft's multiple acts of negligence are a factual cause of the plane's damage.

D Correct. See Restatement (Third) Torts § 26 and illustration 3, upon which this hypothetical is based.

Lilly was a guest at Motel, which was located in a neighborhood where significant violent crime existed. After Lilly returned to her hotel room, Don was able to gain entrance to Lilly's room because the lock on the door was of the simple residential type that could be easily defeated with a credit card. After gaining entrance to Lilly's room, Don sexually assaulted her. Lilly sues Motel, claiming that it was negligent in providing inadequate locks for its guest rooms. May Hotel be held liable in negligence? A Motel may not be held liable in negligence because Don's criminal act was intentional and therefore breaks the chain of causation. B Motel may not be held liable because Don's criminal act was an intervening cause. C Motel may be held liable because Don's act was unforeseeable, unusual, and highly culpable. D Motel may be held liable in negligence because Don's criminal act was foreseeable from its negligence.

D Correct. The initial tortfeasor may be held liable when the criminal act was foreseeable as it was here. This question asks about intervening causes and proximate causation determinations. When an intervening cause is unforeseeable, unusual, or highly culpable, the initial tortfeasor is no longer a proximate cause of the plaintiff's harm.

Insane person erroneously believes that individual is sick and needs help. Insane person believes that individual can be cured by hitting the person over the head with a baseball bat. Insane person hits individual in the head with a baseball bat to cure the person and causes the person serious harm. Did the insane person commit a battery? A No, because the insane person was mistaken in believing the baseball bat would cure the person. B Yes, because the insane person intended no harm but only intended to help. C No, because an insane person cannot have intent. D Yes, because the insane person intentionally caused contact which was harmful or offensive to the person of another.

D Correct. The insane person is liable for intentional torts exactly like a sane person would be and here the insane person acted intentionally to cause contact to the individual which was harmful or offensive.

About two years ago, Client was a patient of Ophthalmologist. Client alleges that during the time that he was Ophthalmologist's patient, Ophthalmologist improperly performed LASIK surgery on Client's eyes. The LASIK surgery was supposed to correct Client's poor vision. Instead, the surgery was performed skillfully, but resulted in blindness in both eyes, a risk about which Client was never informed and a reasonable person would not have known. If Client had known of the risk of blindness, he would not have opted for the surgery. Client hires Attorney Matthew to sue Ophthalmologist for medical malpractice. Regarding the underlying medical malpractice action, which of the following statements concerning the Ophthalmologist's liability is correct? A Ophthalmologist's failure to inform Client of the material risks and available alternatives to LASIK treatment does not constitute a breach of the standard of care in negligence because every patient should know that any eye surgery could lead to blindness. B Ophthalmologist's failure to inform Client of the material risks and available alternatives to LASIK treatment does not constitute a breach of the standard of care in negligence because the reasonable patient should know that any eye surgery could result in blindness. C Ophthalmologist cannot be subject to liability in negligence because he performed the surgery skillfully. D Ophthalmologist's failure to inform Client of the material risks and available alternatives to LASIK treatment is a breach of the standard of care in negligence.

D Correct. This question asks about the informed consent doctrine in negligence law. A physician can be subject to liability in negligence if he fails to inform his patient of the material risks and alternative treatments to the patient's treatment protocol. Failure to inform constitutes a breach of the standard of care in negligence.

Late one night, Pete places his loaded pistol on the coffee table in the family room of his home that he shares with his wife and two young children. The next afternoon Pete's 10-year-old son, Nolan, is spending time in the family room with his friend, Buddy. Nolan picks up the gun and plays with it, accidentally causing it to discharge. Buddy is injured by the gunshot. In a negligence action against Pete brought on behalf of Buddy, can Pete be held liable for negligence? A Pete cannot be held liable in negligence for injuries to Buddy because Pete never intended for Buddy or anyone else to be injured. B Pete cannot be held liable in negligence for injuries to Buddy because Nolan is the person who actually discharged the gun and is therefore the sole proximate cause of Buddy's injuries. C Pete may be held liable in negligence because parents are liable for injuries caused by their children. D Pete may be held liable in negligence because Pete carelessly provided Nolan access to the gun in circumstances in which it was foreseeable that Nolan might use it improperly.

D Correct. When Pete carelessly leaves his gun accessible to his kid, it is foreseeable that one of his kids may misuse it thereby causing an injury. This question asks about conduct that is negligent because of the prospect of improper conduct by another person. The rule is that conduct of the defendant can lack reasonable care if it foreseeably combines with improper conduct of another. Restatement (Third) Torts § 19.

Alice, Bill, and Charles, each acting independently but simultaneously, negligently lean on Polly's car, which is parked at an overlook at the edge of a mountain. Their combined force results in Polly's car rolling over the edge of the cliff and plummeting down the mountain to its destruction. The force exerted by either Alice, Bill, or Charles would have been insufficient alone to propel Polly's car over the edge, but the combined force of any two of them would have been sufficient. In a lawsuit brought by Polly against Alice, Bill, and Charles based on negligence, who is the factual cause of the destruction of Polly's car? A Neither Alice, Bill, nor Charles is a factual cause of the destruction of Polly's car because the force exerted by each of them alone would have been insufficient to move Polly's car down the mountain. B Neither Alice, Bill, nor Charles is a factual cause of the destruction of Polly's car because the combined force of any two of them would have been sufficient. C Two and only two of Alice, Bill, and Charles are a factual cause of the destruction of Polly's car because the combined force of any two of them would have been sufficient. D Alice, Bill, and Charles are each a factual cause of the destruction of Polly's car because their combined force resulted in the destruction Polly's car.

D Correct. When multiple acts combine, each which alone would have been insufficient, each is a factual cause of the harm. Therefore, Alice, Bill, and Charles are each a factual cause of the destruction of Polly's car.

Parker parks his car at the top of a driveway, which is on an incline. Two minutes later, the car rolls down the incline and injures Pedestrian who was on the sidewalk. In suing Parker, Pedestrian seeks to rely on res ipsa loquitur in order to prove Parker's negligence. Which of the following statements about res ipsa loquitur is correct? A Relying on res ipsa loquitur in these circumstances would be improper if there are other possible explanations for why the car rolled. B If able to employ the doctrine of res ipsa loquitur, the jury is required to infer that Parker was negligent. C If permitted to employ the doctrine of res ipsa loquitur, the jury may not infer that Parker was negligent. D If permitted to employ the doctrine of res ipsa loquitur, the jury may infer that Parker was negligent.

D Correct. This question asks about the use of res ipsa loquitur to establish a defendant's negligence. If allowed to use the doctrine of res ipsa loquitur, the jury is permitted but is not required to infer that the defendant was negligent. See Restatement (Third) Torts § 17 and illustration 2, upon which this question was based.

After drinking himself silly at a bar, drunk decides to walk home. Drunk sees a car and decides that it would be fun to scratch the car with his keys. Drunk scratches the car repeatedly, destroying the paint job in the process. The next morning drunk has no recollection of the incident and when he is sued by the car owner for trespass to chattel drunk defends by saying the following, "Hey I did not even know what I was doing, if I was sober I would never have done that." Is drunk's statement a valid defense? A Yes. If drunk cannot remember what he was doing to the car he cannot be liable. B Yes. If drunk was not really in control of himself because of the alcohol he was not acting intentionally. C No. Any harm caused by an intentional tort is compensable in damages. D No. Voluntary intoxication is not a valid defense to intentional tort.

D Correct. Voluntary intoxication is not a defense to intentional tort.

Tracie and Francis were soccer rivals. One day before a game between their respective teams Tracie whispered to Francis, "I am going to beat you down immediately after this 90 minute game is over if you score. Two minutes into the game Francis scored a goal. Immediately after scoring he experienced nervousness and became fearful of what Tracie would do to him after the game. Surprisingly, Francis fainted as a result of the fear, hit his head on the pitch, and had to be carried off unconscious in a stretcher. Should Francis sue for assault and/or battery? A) Francis should sue for assault and battery. B) Francis should sue for assault but not battery. C) Francis should sue for battery but not assault. D) Francis should not sue for assault nor should he sue for battery.

D. Assault is defined as intentionally causing fear or apprehension of imminent harmful or offensive contact. Here, Tracie intentionally caused fear or apprehension of harmful or offensive but the contact was not imminent. The threatened contact was at the beginning of a 90 minute game and therefore was not imminent. Good examples of imminence is swinging a bat at someone's face but stopping an inch short of hitting them, making someone jump, etc. Battery is also not actionable because even though Tracie threatened to cause harmful or offensive contact, the contact that happened was not substantially certain to occur. The facts say it wasn't even predictable because it was surprising that contact occurred.

Law student is angry at Torts professor because professor made student work and professor is always so prepared in class. As professor is walking to his car in his perfectly matching designer shoes student sneaks up behind him and with a Torts casebook, hits professor on the back of his head. Professor slumps unconscious to the ground. The student was so stealthy that the professor never even knew what happened until he regained consciousness. Are the student's action assault and/or battery? A) The students actions are both an assault and battery. B) The students actions are neither an assault nor a battery. C) The students actions are an assault but not a battery. D) The students actions are a battery but not an assault.

D. Battery is intentionally causing contact to the person of another which is harmful or offensive. Clearly the student intended to cause contact to the professor. He did cause contact. Being hit on the back of the head hard enough to lose consciousness is clearly harmful or offensive. So clearly there is a battery. However, there is no assault. Assault is intentionally causing fear or apprehension of imminent harmful or offensive contact. The facts state that the student was so stealthy that the professor did not know what happened until he woke up. His lack of awareness forecloses fear or apprehension.

A commercial aircraft pilot had an argument with her friend. Her friend lives 300 miles from an airport that the commercial pilot usually flies out of. One morning the commercial pilot realized that her flight path for that day would take her really close to the property of her friend. On that day the pilot lifted off from the airport piloting a sleek modern Boeing 787 aircraft. The aircraft climbed to an altitude of 39,000 feet where it could not be seen or heard from people on the ground. At that height and traveling at approximately 550 miles per hour the pilot diverted the plane ever so slightly so it passed directly over her friend's house. Her friend was simply unaware that the aircraft flew over her property as it did so. After the pilot landed she called her friend and said, "I was so angry at you I flew my airplane at 39,000 feet directly over your property." What result if the friend sues the pilot for trespass to land? A The friend will prevail because of the cujus est solum est uque ad coleum doctrine. B The pilot will prevail because the cujus est solum est uque ad coleum doctrine has been overruled. C The friend will not prevail as air travel above 5000 feet is not trespass to land as a matter of law. D The pilot will prevail because the airplane was not in the immediate reaches of the land and did not interfere with the use and enjoyment of the land.

D. Correct. The general restatement rule now is that aircraft overflight can only be considered trespass if the overflight occurs in the immediate reaches of the land and interferes with the use and enjoyment of the land. Here the friend was simply unaware of the aircraft overflight and the plane was at an altitude where it could not be seen or heard from land so it clearly does not satisfy the elements of "immediate reaches" and "interfering with use and enjoyment." The cujus est solum est uque ad coleum doctrine is no longer dispositive in and of itself and so overflights are no longer trespass as a matter of law.

A pointed a gun at B scaring B and without authority or justification, A ordered B into a room and locked the door for 2 hours. B was awake the whole time and worried about what would happen until A released him unharmed. When A initially pointed gun at B, B was standing next to C and C dove for cover thinking the gun was pointed at her. In diving for cover C collided violently with D causing D to spill coffee on E's computer. E subsequently had to pay to have the computer repaired. Who may maintain an action against A? A) Only B but not C, D and E may maintain an action against A B) Only B and C but not D and E may maintain an action against A C) Only B, C and D but not E may maintain an action against A D) B, C, D and E may all maintain actions against A

D. It correctly applies the doctrines of transferred intent and the rule of extended liability in tort. Transferred intent operates whenever the tort intended and the tort committed are within the original writ of trespass (Trespass to Land, Trespass to Chattel, False Imprisonment, Assault and Battery). Whenever a tortfeasor intends any one of the five but ends up committing any other of the five torts then all the torts committed are deemed to have the intent prong satisfied. For example, in the above scenario, A intended to falsely imprison B, but while committing false imprisonment he also caused B to get scared. Therefore, A has committed both an assault and a false imprisonment against B. Additionally, not only does intent transfer between torts but it transfers between people as well. So even though A was intending only to falsely imprison B, he also caused C to get scared and therefore has committed assault against C. Similarly, even though A's only intent was to falsely imprison B, his tortious conduct scared C and resulted in a violent collision with D (harmful or offensive contact), and as a result D has an actionable battery claim against A. Likewise, A's intent to falsely imprison B caused damage to E's chattel (computer), which had to be repaired, and so A is also liable to E for trespass to chattel. Also relevant to this hypothetical is the rule of limitless causation in intentional tort. A tortfeasor is usually liable for all the harm caused by their tortious conduct whether or not the harm was foreseeable. This is an important distinction between intentional tort and negligence. In negligence the tortfeasor is liable only for the foreseeable harm their tortious conduct causes. So in this scenario B, C, D and E may maintain actions against A.

Sam received a text message with a video from an undercover agent pretending to be a gang member. Video showed a distant relative of Sam being severely beaten by other gang members a few days earlier. Distant relative has a viable cause of action for assault and battery against gang members. Upon seeing the video Sam got sick, vomited quite a few times, and was diagnosed with high blood pressure as a result of the video. He also experienced severely disabling emotional distress as a result of the video. May Sam maintain an action for intentional infliction of emotional distress against the gang members based on his reaction to seeing the video? A Probably, because beating someone is considered extreme and outrageous conduct. B Probably, even if the gang members were not substantially certain Sam would see the video. C Probably, not because Sam is not a close relative to the victim of the tortious conduct. D Probably not, because Sam was not sensorily and contemporaneously present during the beating of his distant relative.

D. One of the nuances of bystander recovery pursuant to IIED based on tortious conduct directed at another person is that the bystander must have observed the conduct as it is happening. According to the Dobbs Hornbook § 307 the line is drawn, "against recovery by excluding those who are not present and who have not had an immediate sensory perception of the injury." Here, Sam was not present and he received the video "a few days" after the "primary injury." Therefore, he would not be able to maintain the IIED action based on the tortious conduct directed to his distant relative.


Conjuntos de estudio relacionados

DNA Genes and Proteins (Quiz 10)

View Set

Nurs 2300 Midterm1 Patients with hematologic neoplasms

View Set

Employee Engagement and Retention

View Set

Series 65 - Practice Exam (No Answer Selected)

View Set

Purposive Comm: Midterms (Quizzes)

View Set